weaken
can someone explain how each of the four answer incorrect answer choices actually weaken the answ...
Elizabeth25 on October 4 at 01:31AM
  • October 2000 LSAT
  • SEC1
  • Q17
1
Reply
Why not D
I didn't choose A because the first sentence doesn't read as a general principle to me. A general...
AndrewArabie on April 9, 2023
  • October 2000 LSAT
  • SEC1
  • Q8
4
Replies
Quantifiers
I eliminated A right away because the quantifier in the stimulus, "many" seemed to not line up wi...
JonJay on April 23, 2022
  • October 2000 LSAT
  • SEC1
  • Q16
2
Replies
B
Can someone please explain why B in incorrect. I read the explanation given in the breakdown and ...
Abigail-Okereke on January 19, 2022
  • October 2000 LSAT
  • SEC1
  • Q18
1
Reply
Help
Please explain answer thanks
TheFacu on May 24, 2021
  • October 2000 LSAT
  • SEC1
  • Q20
2
Replies
Why not A?
I can see the difference between A and the correct answer as A uses stronger language, but I can’...
vrgrimm on December 25, 2020
  • October 2000 LSAT
  • SEC1
  • Q3
1
Reply
D
can someone please explain why the answer is D? Does D strengthen the argument somewhat because i...
hfatima1 on August 1, 2020
  • October 2000 LSAT
  • SEC1
  • Q17
1
Reply
Typo in question
The below answer from your team indicated that there is a (major) typo for the correct answer in ...
liz_hassett on August 1, 2020
  • October 2000 LSAT
  • SEC1
  • Q22
1
Reply
Answer Choice B vs C
Hi! I thought C was incorrect because it seems to directly affect the larger animals, not indirec...
DavidW on April 10, 2020
  • October 2000 LSAT
  • SEC1
  • Q15
1
Reply
What’s the main difference between A and B?
This one stumped me. I had it down to A and B and I thought they both were saying the same thing....
Henleys on March 27, 2020
  • October 2000 LSAT
  • SEC1
  • Q14
1
Reply
How does the flaw in "D" match?
"Most people who shop for groceries no more than three times a month buy prepared frozen dinners ...
DDL on December 4, 2019
  • October 2000 LSAT
  • SEC1
  • Q22
1
Reply
Why is answer choice B wrong?
I took it to mean that a person can attend an even without having a permison to attend (without...
Milica Gligic on November 23, 2019
  • October 2000 LSAT
  • SEC1
  • Q6
1
Reply
Why is C incorrect?
It seems like c can be properly inferred from the statement
zgnewquist on September 25, 2019
  • October 2000 LSAT
  • SEC1
  • Q18
2
Replies
Explanation
Why not D?
DanielDePasquale on December 28, 2018
  • October 2000 LSAT
  • SEC1
  • Q16
3
Replies
Help
I was torn between A and D for this question. Can someone please elaborate as to why A is incorrect?
Bly2828 on December 19, 2018
  • October 2000 LSAT
  • SEC1
  • Q23
1
Reply
Help
Please explain how (C) weakens the argument that " the embezzler is one of the actuaries." Of cou...
Batman on December 26, 2014
  • October 2000 LSAT
  • SEC1
  • Q17
2
Replies